Tải bản đầy đủ (.pdf) (52 trang)

Lý thuyết ramsey và một số ứng dụng

Bạn đang xem bản rút gọn của tài liệu. Xem và tải ngay bản đầy đủ của tài liệu tại đây (1.61 MB, 52 trang )

1
MỤC LỤC
Trang
Mục lục 1
Lời nói đầu 2
Chương 1 Kiến thức chuẩn bị 4
Chương 2 Định lí Ramsey 7
§1
Định lí Ramsey trong lí thuyết đồ thị
7
§2
Định lí Ramsey trong tập hợp hữu hạn
15
Chương 3 Ứng dụng của định lí Ramsey 15
§1 Định lí Schur 25
§2 Định lí Erdös–Szekeres 32
§3 Ứng dụng trong giải toán phổ thông 39
Kết luận 51
Tài liệu tham khảo 52



2
LỜI NÓI ĐẦU
Năm 1928, nhà toán học người Anh Frank Plumpton Ramsey đã công bố kết quả
chứng minh của ông trên tạp chí “On a Proplem of Formal logic” trong đó ông đã
chứng minh định lí”Giả sử họ
 
r
S


được phân hoạch thành hai họ các tập
hợp
A

B
,
p

q
là hai số nguyên sao cho
,r p q s 
. Khi ấy tồn tại số nguyên
nhỏ nhất
 
, ,R p q r
chỉ phụ thuộc vào các số
, ,p q r
mà không phụ thuộc vào tập
S
, sao cho nếu
 
, ,s R p q r
thì tồn tại một tập
P
gồm
p
phần tử của
S
, mà tất
cả các tập con

r
phần tử của
P
đều thuộc
A
, hoặc tồn tại một tập
Q
gồm
q
phần
tử của
S
, mà tất cả các tập con
r
phần tử của
Q
đều thuộc
B
”. Định lí trên sau
này được gọi là Định lý Ramsey. Định lí trên đã mở ra một cách tiếp cận mới về
các bài toán tổ hợp nay được gọi là lý thuyết Ramsey.
Trong luận văn này tôi sẽ giới thiệu một số kết quả quan trọng trong lý thuyết
Ramsey và một số ứng dụng của lí thuyết này.
Năm 1916 Issai Schur đã chứng minh rằng “Cho
r
là số tự nhiên,
1r
. Khi đó tồn
tại một số tự nhiên
( )S r

sao cho
( ) N S r
, tập số {1,2,…,N} được tô bởi
r
màu
luôn tồn tại ba số
, ,x y z
có cùng màu sao cho
 x y z
”. Kết quả cơ bản này đã
được tổng quát hóa bởi Richard Rado vào năm 1933.
Định lý Van der Waerden đã được chứng minh vào năm 1927, một năm sớm hơn
so với chứng minh của Ramsey. Van der Waerden đã chứng minh rằng “Cho
k r,

là số tự nhiên
k r, 1
. Khi đó tồn tại một số tự nhiên
k rW( , )
sao cho
N k rW( , ) 
, tập số {1,2,…,
k rW( , )
} được tô bởi
r
màu luôn tồn tại k số được
tô cùng màu lập thành một cấp số cộng”.

3
Năm 1935, Paul Erdös và György Szekeresđã phát biểu giả thuyết (Erdös–

Szekeres, 1935) “ Với mỗi số tự nhiên
3n 
, mọi tập có tối thiểu
 
2
2 1
n
N n

 

điểm trên mặt phẳng ở vị trí tổng quát, đều chứa n điểm là đỉnh của một đa giác
lồi n cạnh.”
Các kết quả trên có thể được chứng minh độc lập với định lý Ramsey, tuy nhiên
sau khi Ramsey công bố kết quả chứng minh của mình thì các bài toán trên đã
được chứng minh lại theo cách ngắn gọn hơn nhờ áp dụng định lý Ramsey.
Luận văn được chia làm ba chương
Chương I Kiến thức chuẩn bị
Chương II Định lý Ramsey
Chương III Ứng dụng của định lý Ramsey.
Luận văn được hoàn thành dưới sự hướng dẫn tận tình của PGS.TS. Tạ Duy
Phượng, Viện Toán học – Viện Khoa học và Công nghệ Việt Nam. Em xin bày tỏ
lòng biết ơn sâu sắc nhất đối với Thầy.
Tôi xin được cảm ơn khoa Toán, khoa Sau Đại học trường Đại học Khoa học Tự
nhiên, Đại học Quốc gia Hà Nội đã quan tâm giúp đỡ, tạo điều kiện thuận lợi cho
tôi thực hiện kế hoạch học tập.

Hà Nội ngày 26 tháng 8 năm 2014



Đinh Hữu Lâm

4
CHƯƠNG 1 KIẾN THỨC CHUẨN BỊ
§1.1 Một số khái niệm cơ bản của lí thuyết đồ thị
Định nghĩa 1.1.1Đồ thị được hiểu là một bộ hai tập hợp hữu hạn : tập hợp đỉnh và
tập hợp cạnh nối các đỉnh này với nhau.
Định nghĩa 1.1.2Mộtđồ thị n đỉnh được gọi là đồ thị đầy đủ nếu hai đỉnh bất kì của
nó có đúng một cạnh nối.
Một đồ thị đầy đủ
n
đỉnh kí hiệu là
n
K
.
Định nghĩa 1.1.3 Một đồ thị gọi được là đồ thị đều bậc t nếu mỗi đỉnh của đồ thị G
có bậc là t.
Định nghĩa 1.1.4 Một đồ thị
n
đỉnh gọi là đồ thị hai màu nếu các cạnh của nó
được tô bởi hai màu xanh hoặc đỏ.
Định nghĩa 1.1.5 Một đồ thị
n
đỉnh gọi là đồ thị
r
màu nếu các cạnh của nó được
tô bởi một trong
r
màu
1 2

, , ,
r
k k k
.
§1.2
Nguyên lí Dirichlet
Định lí Ramsey là một cách mở rộng của Nguyên lí chuồng chim bồ câu (Nguyên lí
Dirichlet): Một tập gồm
s
phần tử được chia thành
n
các tập con phân biệt. Nếu
s n
thì tồn tại ít nhất một tập con chứa nhiều hơn một phần tử.
Nguyên lí Dirichlet được dùng để giải bài toán sau.
Bài toán 1.2.1
Sáu điểm được nối với nhau bởi các đoạn thẳng tô màu đỏ hoặc xanh. Chứng minh
rằng tìm được một tam giác đơn sắc, nghĩa là tam giác có ba cạnh cùng có màu đỏ
hoặc màu xanh.
5
Giải. Kí hiệu một đỉnh là
O
, các đỉnh còn
lại là
, , , , .A B C D E
Theo nguyên lí Dirichlet,
có ít nhất ba trong số các cạnh
, , , ,OA OB OC OD OE
là cùng màu (đỏ hoặc
xanh). Giả sử

, ,OA OB OC
có cùng màu đỏ.
Xét tam giác
ABC
. Nếu có ít nhất một
cạnh, thí dụ,
AB
là màu đỏ, thì tam giác
OAB
có cả ba cạnh là màu đỏ. Nếu tất cả
các cạnh của tam giác
ABC
là màu xanh, thì tam giác
ABC
là tam giác cần tìm.
Như vậy, từ 6 điểm không thẳng hàng, ta có tất cả
3
6
20
C

tam giác, trong đó có ít
nhất một tam giác đơn sắc (các cạnh cùng màu đỏ hoặc xanh).
Câu hỏi Số điểm ít nhất là bao nhiêu để không
có một tam giác đơn sắc?
Trả lời Số điểm ít nhất là 6. Thật vậy, xét năm
điểm
, , , ,A B C D E
với các cạnh
, , , ,AB BC CD DE EA

màu xanh và các cạnh
, , , ,AD AC BE BD CE
màu đỏ. Khi ấy màu của
các cạnh của các tam giác được cho trong hình
bên và bảng sau.

Tam giác
Cạnh xanh

Cạnh đỏ
Tam giác
Cạnh xanh Cạnh đỏ
ABC
AB; BC AC
ADE
AE; DE AD
ABD
AB; AD; BD
BCD
BC;CD BD
F
E
C
B
A
O
6
ABE
AB; AE BE
BCE

BC BE; CE
ACD
CD AD;AC
BDE
DE BD;BE
ACE
AE AC; CE
CDE
CD; DE CE
Như vậy, không có tam giác nào trong số tất cả 10 tam giác có các cạnh cùng màu.
Nói cách khác, trong năm điểm bất kỳ không có một tam giác đơn sắc.
Dưới đây ta cố gắng phát triển tư tưởng của Bài toán 1.2.1 trên ngôn ngữ tập hợp
kết hợp với nguyên lí Dirichlet. Trong nguyên lí Dirichlet, tập gồm
s
phần tử được
chia thành
n
tập với các phần tử riêng rẽ. Nếu
s n
thì tồn tại một tập gồm ít nhất
hai phần tử từ tập cho trước (gồm
s
phần tử). Tập hai phần tử này có thể coi như
một tập chứa hai tập con một phần tử trong cùng một lớp chia.
Trong Bài toán 1.2.1, ta có một tập gồm sáu phần tử. Chúng có thể được coi là sáu
đỉnh của một đồ thị, kí hiệu là
6
K
. Ta có thể coi các tập hai phần tử lấy từ tập các
đỉnh như là các cạnh. Và coi tập ba phần tử (ba đỉnh) như là tam giác. Chia các

cạnh thành hai lớp, là lớp các cạnh màu đỏ và lớp các cạnh màu xanh. Và ta đã chỉ
ra rằng trong đồ thị
6
K
có một tam giác có cả ba cạnh được tô cùng màu đỏ hoặc
cùng màu xanh. Nói cách khác, nếu ta chia tập gồm sáu điểm thành hai lớp các tập
hợp hai điểm, thì có một tập ba điểm nào đó trong tập gồm sáu điểm có mọi tập
con hai điểm nằm trong cùng một lớp chia.
Từ bài toán đơn giản trên, ta đi đến một định lí quan trọng là định lí Ramsey được
trình bày trong chương 2.


7
CHƯƠNG 2 ĐỊNH LÍ RAMSEY
§2.1
Định lí Ramsey trong lí thuyết đồ thị
1. Định lí Ramsey cho đồ thị hai màu
Định lí 2.1.1Cho 2 số tự nhiên p và q. Khi ấy tồn tại số tự nhiên nhỏ nhất
 
,R p q
chỉ phụ thuộc vào các số
,p q
sao cho mọi đồ thị đầy đủ n đỉnh 2 màu,
 
,n R p q
luôn tồn tại một đồ thị con đầy đủ p đỉnh mà tất cả các cạnh được tô
màu xanh hoặc q đỉnh mà tất cả các cạnh được tô màu đỏ.
Bổ đề 2.1.2 1)
 
, 2

R p p
. 2)
 
2,
R q q
. 3)
   
, ,R p q R q p
.
Chứng minh
1) Nếu
 
, 2 1R p p 
ta chọn đồ thị đầy đủ có
1p
đỉnh mà tất cả các cạnh
được tô màu xanh thì đồ thị này không thỏa mãn định lí suy ra
 
, 2 1R p p 
.
Ta xét một đồ thị 2 màu đầy đủ có
p
đỉnh. Nếu tất cả các cạnh được tô màu xanh
thì
p
đỉnh này thỏa mãn định lí. Nếu tất cả các cạnh không được tô màu xanh thì
tồn tại một cạnh được tô màu đỏ giả sử là
AB
thì 2 đỉnh
,A B

thỏa mãn định lí suy
ra
 
, 2
R p p
. Vậy
 
, 2
R p p
.

2) Chứng minh tương tự ta có
 
2,
R q q
.
3) Do vai trò như nhau của 2 màu xanh ,đỏ nên ta có
   
, , .R p q R q p

Bổ đề 2.1.3
     
, 1, , 1
R p q R p q R p q
   
.
Chứng minh
Đặt
   
1, , 1

n R p q R p q
   
xét đồ thị 2 màu đầy đủ n đỉnh kí hiệu là
n
K
.
Gọi
A
là một đỉnh bất kì thuộc đồ thị.
8
Kí hiệu:
A
B
= {Tập tất cả các đỉnh thuộc
n
K
mà cạnh nối nó với A có màu xanh}

A
R
= {Tập tất cả các đỉnh thuộc
n
K
mà cạnh nối nó với A có màu đỏ}


n
K
là đồ thị đầy đủ nên
1  

A A
B R n
.
Nếu
 
1, 
A
B R p q

 
, 1 
A
R R p q
thì
2  
A A
B R n
vô lí.
Vậy
 
1, 
A
B R p q
hoặc
 
, 1 
A
R R p q
.


Giả sử
 
1, 
A
B R p q
suy ra hoặc tồn tại
1p
đỉnh mà tất cả các cạnh có màu
xanh thì
1p
đỉnh này cùng với
A
tạo thành đồ thị có
p
đỉnh mà tất cả các cạnh
có màu xanh hoặc tồn tại
q
đỉnh mà tất cả các cạnh có màu đỏ .Vậy khi
   
1, , 1
n R p q R p q
   
thì luôn tồn tại
p
đỉnh mà tất cả các cạnh có màu
xanh hoặc tồn tại
q
đỉnh mà tất cả các cạnh có màu đỏ tức là
     
; 1, , 1

R p q n R p q R p q
    
.
Chứng minh Định lí 2.1.1
Ta sẽ chứng minh Định lí 2.1.1 bằng cách quy nạp theo
,p q
.
Theo Bổ đề 2.1.2 ta có
 
, 2
R p p
; 2)
 
2,
R q q
.
Giả sử
 
, 1
R p q


 
1,R p q
là tồn tại theo Bổ đề 2.1.3 ta có
     
, 1, , 1
R p q R p q R p q
   
suy ra định lí được chứng minh.

2. Số Ramsey
Định nghĩa 2.1.4 Số
 
,R p q
trong Định lí 2.1.1 được gọi là số Ramsey.
9
Ta thấy bài toán 1.2.1 là một trường hợp đặc biệt của định lí Ramsey lúc này
 
3,3 6
R

.
Bài toán 2.1.5 Chứng minh rằng
 
3,4 9
R

.
Chứng minh
Hình bên là một đồ thị gồm 8 đỉnh
8
K
có các cạnh được tô màu đỏ hoặc xanh (đỏ–
đường nét liền; xanh–đường nét đứt) không chứa đồ thị con
3
K
có tất cả các cạnh
màu xanh, cũng không chứa đồ thị con
4
K

nào có tất cả các cạnh màu đỏ. Như vậy
 
3,4 8
R

.


Ta phải chứng minh rằng trong đồ thị hai màu
9
K
thì luôn tồn tại một đồ thị con
3
K
có tất cả các cạnh màu xanh hoặc một đồ thị con
4
K
có tất cả các cạnh màu đỏ.
Gọi
1
x
là một đỉnh nào đó của
9
K
. Nối
1
x
với tám đỉnh còn lại thì theo nguyên lí
Dirichlet phải có tối thiểu bốn đỉnh nối với
1

x
bởi các cạnh hoặc màu đỏ hoặc màu
xanh.
1
2
3
4
5
6
7
8
1
7
6
5
4
8
3
6

10
Trường hợp 1:Có ít nhất bốn đỉnh được đánh số là
2 3 4 5
, , ,x x x x
nối với
1
x
bởi các
cạnh màu xanh.
Nếu tồn tại một cặp hai điểm trong số bốn đỉnh

2 3 4 5
, , ,x x x x
, thí dụ,
2 3
,x x
được
nối với nhau bởi cạnh màu xanh, thì ta có đồ thị con
 
3 1 2 3
, ,K x x x
có tất cả các
cạnh màu xanh. Nếu không tất cả các cạnh tạo bởi bốn điểm
2 3 4 5
, , ,x x x x
phải
đều là màu đỏ, khi ấy ta lại có đồ thị con
 
4 2 3 4 5
, , ,K x x x x
có tất cả các cạnh
màu đỏ.
Trường hợp 2:Có nhiều nhất ba đỉnh nối với
1
x
bởi các đoạn màu xanh, nghĩa là
có ít nhất năm đỉnh được nối với
1
x
bởi các đoạn màu đỏ.
Nếu tất cả các đỉnh được nối với 8 đỉnh còn lại bởi năm cạnh màu đỏ và ba cạnh

màu xanh.Bây giờ ta xét đồ thị con đơn sắc (monochromatic subgraphs). Trong đồ
thị con đơn sắc màu xanh, có tất cả 9 đỉnh, mỗi đỉnh có bậc 3 (có ba cạnh xuất
phát từ một đỉnh có màu xanh). Suy ra số cạnh màu xanh là
9.3 27
2 2

(vì mỗi cạnh
được tính 2 lần).Vô lí.
Vậy phải tồn tại đồ thị
6
K
được tạo bởi 6 đỉnh, kí hiệu là
2 3 4 5 6 7
, , , , ,x x x x x x
, được
nối với
1
x
bởi các đoạn màu đỏ.
Đồ thị
6
K
có các cạnh đều được tô màu đỏ hoặc màu xanh. Từ Bài toán 1.6 tồn tại
tam giác có ba cạnh màu đỏ hoặc ba cạnh màu xanh. Nếu tam giác này có ba cạnh
màu xanh thì nó cũng là tam giác có ba cạnh màu xanh của
9
K
. Còn nếu tồn tại
một tam giác, kí hiệu là
2 3 4

x x x
có ba cạnh màu đỏ.Khi đó vì
2 3 4
, ,x x x
được nối với
1
x
bởi các cạnh màu đỏ nên đồ thị gồm các đỉnh
 
4 2 3 4 1
, , ,K x x x x
có tất cả bốn
cạnh màu đỏ.

11
Vậy trong mọi trường hợp thì đồ thị 2 màu
9
K
luôn tồn tại một đồ thị con
3
K

tất cả các cạnh màu xanh hoặc một đồ thị con
4
K
có tất cả các cạnh màu đỏ.
Suy ra
 
3,4 9
R


.
Vậy
 
3,4 9
R

.
Bài toán 2.1.6 Chứng minh rằng
 
3,5 14
R

.

Chứng minh


Theo Bổ đề 2.1.3 ta có
     
3,5 2,5 3,4 5 9 14
R R R
    
.
Mặt khác, hình bên là một đồ thị gồm 13 đỉnh
13
K
có các cạnh được tô màu đỏ
hoặc xanh (đỏ – đường nét liền; xanh – đường không nối hai đỉnh bất kì ) không
chứa đồ thị con

3
K
có tất cả các cạnh màu đỏ, cũng không chứa đồ thị con
5
K
nào
có tất cả các cạnh màu xanh. Như vậy
 
3,5 13
R

.
Vậy
 
3,5 14
R

.
1
2
3
4
5
6
7
11
8
9
10
13

12
10
9
8
7
6
5
4
3
2
1
13
12
11

12
Bài toán 2.7 Chứng minh rằng
 
4,4 18
R

.
Chứng minh









Theo Bổ đề 2.1.3 ta có
     
4,4 3,4 4,3 9 9 18
R R R
    
.
Mặt khác hình bên là một đồ thị gồm 17 đỉnh
17
K
có các cạnh được tô màu đỏ
hoặc xanh (đỏ–đường nét liền; xanh–đường nét đứt) không chứa đồ thị con
4
K

tất cả các cạnh màu xanh, cũng không chứa đồ thị con
4
K
nào có tất cả các cạnh
màu đỏ. Như vậy
 
4,4 17
R

.
Vậy
 
4,4 18
R


.


1
2
3
17
16
15
14
13
12
11
10
4
5
6
7
8
9
1
2
3
4
5
6
7
8
9
10

11
12
13
14
15
16
17

13
3. Định lí Ramsey tổng quát
Định lí 2.1.8 Cho r số tự nhiên p
1
, p
2
, , p
r
. Khi ấy tồn tại số tự nhiên nhỏ
nhất
 
1 2
, , ,
r
R p p p
chỉ phụ thuộc vào các số p
1
, p
2
, , p
r
sao cho mọi đồ thị đầy đủ

n đỉnh r màu với
 
1 2
, , ,
r
n R p p p
luôn tồn tại một đồ thị con đầy đủ p
1
đỉnh mà
tất cả các cạnh được tô màu k
1
hoặc p
2
đỉnh mà tất cả các cạnh được tô màu k
2

hoặc hoặc p
r
đỉnh mà tất cả các cạnh được tô màu k
r
.
Chứng minh
Ta sẽ chứng minh Định lí 2.1.8 bằng phép quy nạp theo r.
Với
 
1 1
1:
 r R p p

Với

2r
: Đây chính là Định lí 2.1.1
Giả sử Định lí đúng đến
1r
. Ta sẽ chứng minh định lí đúng đến
r
.
Theo giả thiết tồn tại
 
1 1
, ,
r
q R p p


sao cho mọi đồ thị đầy đủ
q
đỉnh
1r
màu
luôn tồn tại một đồ thị con đầy đủ
1
p
đỉnh mà tất cả các cạnh được tô màu
1
k

hoặc
2
p

đỉnh mà tất cả các cạnh được tô màu
2
k
hoặc
1r
p
đỉnh mà tất cả các
cạnh được tô màu
1r
k
.
Ta tô màu
r
k
là màu đỏ còn các màu
1
k
, ,
1r
k
là màu xanh. Theo Định lí 2.1.1
tồn tại số
 
,
r
n R q p
sao cho mọi đồ thị đầy đủ n đỉnh hai màu luôn tồn tại một
đồ thị con đầy đủ
q
đỉnh mà tất cả các cạnh được tô màu xanh hoặc

r
p
đỉnh mà
tất cả các cạnh được tô màu đỏ.
Vậy luôn tồn tại số tự nhiên
 
1 2
, , ,
r
R p p p
sao cho mọi đồ thị đầy đủ n đỉnh

14
r màu,
 
1 2
, , ,
r
n R p p p
luôn tồn tại một đồ thị con đầy đủ
1
p
đỉnh mà tất cả
các cạnh được tô màu
1
k
hoặc
2
p
đỉnh mà tất cả các cạnh được tô màu

2
k
hoặc
r
p
đỉnh mà tất cả các cạnh được tô màu
r
k
.
Bài toán 2.9 Chứng minh rằng
 
3,3,3 17
R

.
Chứng minh

(Đồ thị Clebsch)
Ta thấy trên đồ thị Clebsch có 16 đỉnh các cạnh được tô bởi 3 màu xanh, đỏ, vàng
nhưng không có tam giác nào có 3 cạnh cùng màu. Suy ra
 
3,3,3 16
R

.

Bấy giờ ta xét một đồ thị đầy đủ 17 đỉnh. Các cạnh của đồ thị được tô bởi một
trong ba màu xanh, đỏ, vàng. Ta cần chứng minh trong đồ thị tồn tại ba đỉnh mà
các cạnh được nối với nhau bởi cùng một màu.
Kí hiệu một đỉnh là A. Vì A nối với 16 đỉnh còn lại bởi ba màu nên theo nguyên tắc

Dirichlet tồn tại 6 đỉnh nối với A bởi cùng một màu. Giả sử sáu đỉnh là B, C, D, E,
F, G nối với A bởi màu xanh.
Nếu trong sáu đỉnh có hai đỉnh nối với nhau bởi màu xanh, giả sử là B,C thìba đỉnh
A,B,C được nối với nhau cùng một màu.

15
Nếu trong sáu đỉnh không có hai đỉnh nào nối với nhau bởi màu xanh. Suy ra trong
6 đỉnh B,C,D,E,F,G các cạnh được nối với nhau bởi hai màu đỏ và vàng theo bài
toán 1.2.1 tồn tại ba đỉnh mà các cạnh được nối với nhau bởi một màu.
Vậy trong 17 đỉnh luôn tồn tại ba đỉnh mà các cạnh được nối với nhau bởi cùng
một màu.
Vậy
 
3,3,3 17
R

.
§2.2
Định lí Ramsey trong tập hợp hữu hạn
1. Định lí Ramsey
Giả sử
S
là tập hợp gồm
s
phần tử. Kí hiệu
 
r
S

là họ tất cả các tập con của

S
, mỗi tập có đúng
r
phần tử,
1r 
. Ta nói họ
 
r
S

được phân hoạch thành
hai họ các tập hợp
A

B
nếu
A

B
khác rỗng và thỏa mãn điều kiện:
 
 
   
;
r
S A B A B
.
Định lí 2.2.1 (Ramsey, 1930) Giả sử họ
 
r

S

được phân hoạch thành hai họ các
tập hợp
A

B
;
p

q
là hai số nguyên sao cho
,r p q s 
. Khi ấy tồn tại số
nguyên nhỏ nhất
 
, ,R p q r
chỉ phụ thuộc vào các số
, ,p q r
mà không phụ thuộc
vào tập
S
, sao cho nếu
 
, ,s R p q r
thì tồn tại một tập
P
gồm
p
phần tử của

S
,
mà tất cả các tập con
r
phần tử của
P
đều thuộc
A
, hoặc tồn tại một tập
Q
gồm
q
phần tử của
S
, mà tất cả các tập con
r
phần tử của
Q
đều thuộc
B
.
Số
 
, ,R p q r
tìm được trong Định lí 2.2.1 được gọi là số Ramsey. Các số Ramsey
rất được quan tâm vì có nhiều ứng dụng, nhưng rất khó tính được nó. Ta mới chỉ
tính được một vài số Ramsey nhỏ. Thậm chí rất khó tìm ra các công thức đánh giá
các số Ramsey.

16

Trong trường hợp
2r 
, ta có thể phát biểu lại Định lí 2.1 dưới ngôn ngữ của lí
thuyết đồ thị như sau. Giả thiết rằng
 
, ,2s R p q
và các cạnh của đồ thị
s
K
đã
được tô bởi hai màu đỏ hoặc xanh. Chia các cạnh của đồ thị thành hai lớp: lớp
A

gồm các cạnh màu đỏ và lớp
B
gồm các cạnh màu xanh. Định lí 2.1 nói rằng, đồ
thị
s
K
phải chứa đồ thị con
p
K
có tất cả các cạnh màu đỏ hoặc chứa đồ thị con
q
K

có tất cả các cạnh màu xanh.
Kết quả của Bài toán 1.6có thể viết lại thành:
 
3,3,2 6

R

.
Bổ đề 2.2.2 1)
  ( , ,1) 1R p q p q
; 2)
( , , )R r q r q
; 3)
( , , )R p r r p
.
Chứng minh
1) Trường hợp
1r 
đưa ta trở về nguyên lí Dirichlet. Giả sử
S
là tập gồm
s

phần tử. Vì
1r 
nên có tất cả
s
tập con một phần tử. Chia
s
tập con một phần
tử này thành hai lớp.
Nếu
2s p q  
thì
   

2 1 1
s p q p q
      
nên ta có thể chia tập hợp
S

thành hai lớp, lớp thứ nhất chứa nhiều nhất
1p 
phần tử và lớp thứ hai chứa nhiều
nhất
1q 
phần tử. Nghĩa là trong cách chia này, ta không có tập nào chứa
p
hoặc
q
phần tử cả. Do đó
  ( , ,1) 2R p q p q
.
Nếu
s p q 
thì hoặc là lớp thứ nhất chứa
1
k
phần tử với
1
k p
, hoặc nếu lớp
thứ nhất chứa
1
k

phần tử với
1
k p
thì lớp thứ hai chứa
2
k
phần tử với
 
2 1
k s k p q p q     
. Nghĩa là hoặc lớp thứ nhất chứa tối thiểu
p
phần tử,
hoặc lớp thứ hai chứa tối thiểu
q
phần tử.
Nếu
1s p q  
thì mọi phép chia tập hợp
S
thành hai lớp thì hoặc lớp thứ nhất
chứa
p k p 
phần tử,
0k 
và lớp thứ hai chứa
1q k 
phần tử; hoặc lớp thứ
nhất chứa
1p k 

phần tử và lớp thứ hai chứa
q k q 
phần tử.

17
Do đó
  ( , ,1) 1R p q p q
.
2) Trường hợp
p r
. Số phần tử
s
của tập hợp
S
phải ít nhất là bao nhiêu để ta
có thể bảo đảm chắc chắn rằng trong
S
hoặc là tồn tại tập
P
gồm
r
phần tử của
S
mà tất cả các tập con
r
phần tử của
P
nằm trong
A
hoặc là tồn tại tập

Q
gồm
q
phần tử của
S
mà tất cả các tập con
r
phần tử của
Q
nằm trong
B
, trong đó
A


B
là một phân hoạch của
 

r
S
, tức là
A

B
là họ các tập con
r
phần tử và
 
 

   
;
r
S A B A B
?

A  
nên
A
phải chứa ít nhất một tập gồm
r
phần tử. Do
p r

r q s 

nên bắt buộc
s q
(để tin tưởng chắc chắn rằng
S
chứa tập
q
phần tử). Do đó
( , , )R r q r q
.
3) Trường hợp
q r
. Chứng minh tương tự trường hợp
p r
ta có

( , , )R p r r p
.
Bổ đề 2.2.2 được chứng minh.
Chứng minh Định lí 2.2.1
Theo Bổ đề 2.2.2 thì Định lí 2.2.1 đúng cho
1r 
. Ta chứng minh qui nạp theo
r
.
Giả thiết rằng định lí đúng cho
1r 
. Bây giờ ta lại sử dụng qui nạp theo
p q
,
bằng cách sử dụng khẳng định 2) và 3) của Bổ đề 2.2.2. Như vậy, theo giả thiết qui
nạp, các số
 
1
1, ,p R p q r 

 
1
, 1,q R p q r 
tồn tại. Ta khẳng định rằng
  
1 1
( , , ) ( , , 1) 1
R p q r R p q r
. Để chứng minh điều này, ta xét tập
S

gồm
  
1 1
( , , 1) 1
s R p q r
phần tử. Giả sử họ
 
r
S

các tập con
r
phần tử của
S
được
tô bởi hai màu: đỏ và xanh. Tương tự như trong chứng minh Bài toán 1.2.1 (cho
6
K
), ta lấy ra một phần tử của
S
và kí hiệu nó là
a
. Bây giờ ta xác định các tập
con tô màu
1r 
phần tử của tập
 
: \S S




như sau:

18
Tập
X S



1r 
phần tử được tô màu trùng với màu của
 
X a
.

S
có số phần tử là
  
1 1
( , , 1) 1
s R p q r
nên
S

có số phần tử
1 1
( , , 1).

 
s R p q r


Theo giả thiết qui nạp, hoặc
S

chứa tập con
A

1
p
phần tử sao cho mọi tập
con
1r 
của nó phải là màu đỏ hoặc
S

chứa tập con
B

1
q
phần tử sao cho mọi
tập con
1r 
của nó phải là màu xanh. Không hạn chế tổng quát, giả sử xảy ra
trường hợp 1, tức là
A

1
p
phần tử sao cho mọi tập con

1r 
của nó có màu đỏ.
Bởi vì
 
1
1, , p R p q r
nên có thể có hai khả năng.
Trường hợp 1 Hoặc
A
có tập con
Q
gồm
q
phần tử, mọi tập con
r
phần tử của
nó có màu xanh. Bởi vì
Q
cũng là tập con
q
phần tử của
S
nên
S
chứa tập con
Q
gồm
q
phần tử, mọi tập con
r

phần tử của nó màu xanh. Như vậy, ta có khẳng
định của định lí.
Trường hợp 2 Hoặc
A
có tập con
A

gồm
1p 
phần tử, mọi tập con
r
phần tử
của nó có màu đỏ. Trong trường hợp này tập con
 
:P A


 
cũng có tính chất
đó, vì
A A


, tức là
P

p
phần tử, mọi tập con
r
phần tử của nó có màu đỏ.

Nhưng
P
cũng là tập con của
S
nên
P
chính là tập cần tìm. Và ta có khẳng định
của định lí.
Trường hợp
S

chứa tập con
B

1
q
phần tử sao cho mọi tập con
1r 
của nó
phải là màu xanh được chứng minh hoàn toàn tương tự.
Điều đó chứng tỏ rằng,
  
1 1
( , , ) ( , , 1) 1
R p q r R p q r
.
Định lí Ramsey được chứng minh.


19

2. Định lí Ramsey tổng quát
Ở hai mục trên ta đã thấy, Định lí Ramsey có thể phát biểu trên ngôn ngữ chia tập
hợp thành hai lớp hoặc ngôn ngữ đồ thị hai màu. Có thể đặt câu hỏi, có thể tổng
quát hóa Định lí Ramsey khi tập hợp được chia thành nhiều lớp hoặc trên ngôn ngữ
đồ thị nhiều màu không?
Định lí 2.2.3(Định lí Ramsey tổng quát)Giả sử
S
là tập hợp gồm
s
phần tử và
 
r
S

là họ tất cả các tập con
r
phần tử của
S
,
1r 
. Giả sử rằng chúng ta có
cách phân hoạch tập hợp
 
1 2

r n
S A A A

   
, sao cho mọi lớp

i
A
khác
trống và mỗi tập con
r
phần tử của
S
thuộc vào chỉ một tập
i
A
(
i j
A A
  
với
mọi
i j
). Giả sử
1 2
, , ,
n
k k k
là các số nguyên sao cho
i
r k s 
với mọi
1,2, ,i n
. Khi ấy tồn tại số nguyên
 
1 2

, , , ;
n
R k k k r
, chỉ phụ thuộc vào
1 2
, , , ,
n
n k k k

r
, mà không phụ thuộc vào tập
S
, sao cho nếu
 
1 2
, , , ;
n
s R k k k r
thì tồn tại một tập con
i
P
gồm
i
k
phần tử của
S
, mà mọi tập
con
r
phần tử của

i
P
thuộc vào tập
i
A
với tối thiểu một giá trị
i
,
1 i n 
.
Chứng minh
Ta sẽ chứng minh theo qui nạp với
2n 
làm điểm xuất phát. Giả sử định lí đã
được chứng minh cho phép chia tập hợp
 
r
S

vào
1n 
hoặc ít hơn các họ tập
hợp. Xét phép chia
 
 
1 2 1

r n n
S A A A A



    
.
Giả sử
 
1 2 1
, , , ;
n
R k k k r



,
 
, ;
n
s R k r



S
là tập gồm
s
phần tử. Khi ấy
hoặc là
S
chứa tập con
n
P
gồm

n
k
phần tử, mọi tập con
r
phần tử của nó thuộc
n
A
. Trong trường hợp này định lí được chứng minh. Hoặc là
S
chứa tập con
T

gồm

phần tử, mọi tập con
r
phần tử của nó thuộc tập
1 2 1

n
A A A

  
. Theo

20
cách chọn

và theo qui nạp, tập
T

phải chứa ít nhất một tập con
i
P
gồm
i
k
phần
tử, mọi tập con
r
phần tử của nó thuộc họ
i
A
với một giá trị
i
nào đó,
1 1i n  
.
Nhưng
i
P
cũng là tập con của tập
S
, do đó trong mọi trường hợp,
S
chứa tập một
tập con
i
P
gồm
i

k
phần tử, mọi tập con
r
phần tử của nó thuộc họ
i
A
với một giá
trị
i
nào đó,
1 i n 
. Định lí được chứng minh.
Trong rất nhiều áp dụng, ta quan tâm đến trường hợp
1 2

n
k k k k   
. Để đơn
giản, ta kí hiệu
 
 
1 2
, , , ; : ;
n n
R k k k r R k r
.
3. Một số công thức đánh giá số Ramsey
Định lí 2.2.1 và Định lí 2.2.3 cho ta biết sự tồn tại số Ramsey
 
, ;R p q r


 
1 2
, , , ;
n
R k k k r
. Tuy nhiên, hai định lí này không cho ta biết cách tính số Ramsey.
Nói chung không có công thức tính số Ramsey. Vì vậy người ta cố gắng đi tìm các
công thức đánh giá số Ramsey. Các đánh giá này hiện nay cũng chưa nhiều. Mục
này trình bày một số giá trị của số Ramsey và một số công thức đánh giá số
Ramsey.
Định lí 2.2.4Với mọi số nguyên
, 2p q 
ta có
 
1
2
, ;2
p
p q
R p q C

 

.
Chứng minh Đặt
n p q 
. Ta sẽ chứng minh bằng qui nạp theo
n
. Để bắt đầu

qui nạp, ta nhắc lại công thức từ Bổ đề 2.1.2:
 
2 1
2 2 2
2,2;2 2R C

 
 
. Vì
, 2p q 
nên
4n p q  

4n 
khi và chỉ khi
2p q 
. Do đó đánh giá trong định lí cũng
đúng cho
4n 
.
Theo Bổ đề 2.2.2, ta có
 
2 1
2 2
2, ;2
q
R q q C

 
 


 
1
2 2
,2;2
p
p
R p p C

 
 
. Như vậy,
đánh giá trong định lí cũng đúng nếu một trong hai số
p
hoặc
q
bằng 2.

21
Bây giờ, không hạn chế tổng quát, ta có thể giả thiết rằng
3, 3p q 
và theo giả
thiết qui nạp theo
n
, đánh giá trong định lí đúng cho mọi số nguyên
2
p




2
q


, trong đó
p q n
 
 

4n 
.
Giả sử
p

q
là các số nguyên sao cho
p q n 
. Áp dụng giả thiết qui nạp cho
1,
p p q q
 
  
ta được
 
2
3
1, ;2
p
p q
R p q C


 
 
.
Do
 
   
 
   
 
   
 
 
   
2 1
3 3
1
2
3 ! 3 !
2 ! 1 ! 1 ! 2 !
3 ! 2 !
1 1 .
1 ! 1 ! 1 ! 1 !
 
   

 
   
  
   

   
     
   
p p
p q p q
p
p q
p q p q
C C
p q p q
p q p q
p q C
p q p q

nên ta chỉ còn phải chứng minh rằng
     
, ;2 1, ;2 , 1;2
R p q R p q R p q
   
.
Từ đây ta có
 
2 1 1
3 3 2
, ;2
p p p
p q p q p q
R p q C C C
  
     

  
.
Đây chính là điều phải chứng minh.
Giả sử
m
K
là một đồ thị đầy đủ (giữa hai đỉnh bất kì có đúng một cạnh nối) có
1
2
p
p q
m C

 

đỉnh. Ta cần chứng minh rằng
m
K
phải chứa hoặc đồ thị con
q
K
có tất
cả các cạnh màu xanh, hoặc đồ thị con
p
K
có tất cả các cạnh màu đỏ, hoặc cả hai.
Do
3p 
nên
1 1q  

;
2 2q  
; ;
2 1q p p   
.
Vì vậy
 
   
   
 
1
2
2 ! . 1 2
1 ! 1 ! 1.2 1
p
p q
p q q q q p
m C q
p q p

 
    
   
  
.

22
Suy ra, nếu mọi cạnh của đồ thị
m
K

là màu xanh, thì
m
K
phải chứa đồ thị con
q
K

có tất cả các cạnh màu xanh và ta có điều cần chứng minh. Nếu
m
K
chứa một
cạnh màu đỏ thì có tối thiểu hai đỉnh được nối với nhau bởi một cạnh màu đỏ. Có
hai khả năng.
Trường hợp 1: Giả sử
x
là đỉnh của đồ thị
m
K
mà có tối thiểu
 
1, ;2s R p q 

đỉnh
1 2
, , ,
s
x x x
nối với nó bởi các cạnh màu đỏ. Xét đồ thị con
s
K

tạo bởi các
đỉnh ấy. Bởi vì các cạnh của đồ thị
s
K
được tô màu đỏ hoặc xanh, nên nó chứa
hoặc là đồ thị con
q
K
có tất cả các cạnh màu xanh, khi ấy định lí được chứng
minh; hoặc hoặc là
s
K
chứa đồ thị con
1p
K

có tất cả các cạnh màu đỏ. Kí hiệu tập
các đỉnh của
1p
K


 
 
1 2 1 1 2
, , , , , ,
p s
y y y x x x



. Khi ấy các đỉnh
1 2 1
, , , ,
p
x y y y

tạo thành đồ thị con
p
K
có tất cả các cạnh màu đỏ và định lí đúng.
Trường hợp 2: Giả sử rằng không có đỉnh nào của đồ thị
m
K
mà có
 
1, ;2R p q

đỉnh được nối với nó bởi các cạnh màu đỏ.

   
2 1 1
3 3 2
1, ;2 , 1;2
p p p
p q p q p q
R p q R p q C C C
  
     
     


nên một đỉnh
x
nào đó của
m
K
có tối thiểu
 
, 1;2
t R p q
 
cạnh màu xanh xuất
phát từ nó và nối với các đỉnh
1 2
, , ,
t
x x x
. Như trong Trường hợp 1, ta xét đồ thị
t
K
của các đỉnh ấy. Đồ thị
t
K
chứa hoặc đồ thị con
p
K
có tất cả các cạnh màu đỏ,
khi ấy định lí được chứng minh, hoặc
t
K
chứa đồ thị con

1q
K

có tất cả các cạnh
màu xanh. Giả sử tập các đỉnh của
1q
K


 
 
1 2 1 1 2
, , , , , ,
q t
y y y x x x


. Khi ấy
các đỉnh
1 2 1
, , , ,
p
x y y y

tạo thành đồ thị con có tất cả các cạnh màu xanh và định lí
cũng được chứng minh trong trường hợp này.

23
Tất cả các trường hợp đã được xét. Định lí chứng minh xong.
Nhận xét Nếu

2p 
hoặc
2q 
hoặc
3p q 
thì dấu bằng xảy ra trong đánh
giá của Định lí 2.1.1. Tuy nhiên, nếu
3q 

4p 
thì ta có
 
3,4;2 9
R

(xem
Bài toán 1.2.1), trong khi đó
1
2
10
p
p q
C

 

.
Định lí 2.2.4Với mọi số nguyên
, 2s t 
ta có

   
 
 
 
2;2 ;2 1 ;2 1 1
n n n
R st s t R s R t
      
.
Chứng minh Đặt
 
;2 1
n
p R s
 

 
;2 1
n
q R t
 
. Xét các đồ thị đầy đủ
p
K

q
K
với các đỉnh tương ứng là
1 2
, , ,

p
x x x

1 2
, , ,
q
y y y
. Tô các cạnh của
p
K

q
K
bởi
n
màu
1 2
, , ,
n
C C C
sao cho
p
K
không chứa
s
K
đồ thị đơn sắc, và
q
K


không chứa đồ thị
t
K
đơn sắc. Cách tô màu này là có thể theo Định lí Ramsey và
định nghĩa của các số
p

q
(vì
   
;2 1 ;2
n n
p R s R s  
).
Bây giờ giả sử
pq
K
là một đồ thị đầy đủ với các đỉnh là
ij
z
với
1,2, ,i p

1,2, ,j q
. Tô màu các cạnh của đồ thị
pq
K
như sau.
Giả sử cạnh là
ij gh

z z
.
1) Nếu
i g
thì tô màu
ij gh
z z
giống màu của cạnh
j h
y y
trong
q
K
.
2) Nếu
i g
thì tô màu
ij gh
z z
giống màu của cạnh
i g
x x
trong
p
K
.
Đặt
2r st s t   
, khi ấy
r pq

. Xét đồ thị con
r
K
của
pq
K
. Ta sẽ chỉ ra rằng
r
K
không thể là đồ thị đơn sắc. Giả sử ngược lại,
r
K
là đồ thị đơn sắc, thí dụ, tất
cả các cạnh của nó được tô một màu
1
C
. Xuất hiện hai khả năng.

24
1) Có tối thiểu
s
giá trị khác nhau của
i
, thí dụ,
1,2, ,i s
mà đỉnh
ij
z
nằm trong
r

K
. Vì
ij
z

gj
z
có chỉ số
i g
nên các cạnh
ij gj
z z
có màu cùng màu với các cạnh
i g
x x
trong
p
K
. Do
r
K
là đơn sắc nên
ij gj
z z
cùng màu, suy ra
i g
x x
cùng màu.
Chứng tỏ
p

K
chứa một đồ thị con đơn sắc
s
K
. Vô lí.
2) Tồn tại tối đa
1s 
giá trị khác nhau của
i
mà các đỉnh
ij
z
là các đỉnh của
r
K
.
Nếu chỉ tồn tại tối đa
1t 
giá trị khác nhau của
j
sao cho các đỉnh
ij
z
là đỉnh của
r
K
thì
r
K
có tối đa

  
1 1 1 1s t st s t r       
đỉnh. Nhưng
r
K

r
đỉnh.
Điều mâu thuẫn này suy ra rằng có tối thiểu một giá trị của
i
để
ij
z
là đỉnh của
r
K

cho tối thiểu
t
giá trị của
j
. Thay của
p
thành
q

s
thành
t
trong trường hợp

1, ta suy ra
q
K
chứa một đồ thị đơn sắc
t
K
. Một lần nữa lại mâu thuẫn.
Như vậy,
pq
K
không chứa đồ thị đơn sắc
r
K
. Định lí được chứng minh.
Định lí 2.2.5Ta có các đẳng thức và bất đẳng thức sau đây:
1)
(3,5;2) 14R
; 2)
(3,6;2) 18R
; 3)
(3,7;2) 23R
;
4)
(3,8;2) 28R
5)
(3,9;2) 36R
; 6)
(4,4;2) 18R
;
7)

(4,5;2) 25R
; 8)
   
3
3;2 3,3,3;2 17
R R
 
;
9)
 

  
1 2 2
2
2 2
2 ( , ;2) 2
p
p p
p
R p p C
(Erdős

Szekeres, 1935; Erdős, 1947);
10)
 
2
2 ( , ;3) 2
n
bn c
R n n

với hai hằng số
b

c
nào đó.




25
Bảng dưới đây ghi lại một số kết quả về số Ramsey trong trường hợp hai màu

3 4 5 6 7 8 9 10
3 6 9 14 18 23 28 36 (40,43)
4 18 25 (35,41) (49,61) (53,84) (69,115)

(80,149)

5 (43,49)

(58,87) (80,143)

(95,216)


6 (102,165)














p

q

×